Stay ahead of learning milestones! Enroll in a class over the summer!

G
Topic
First Poster
Last Poster
Geometry
youochange   3
N 21 minutes ago by Double07
m:}
Let $\triangle ABC$ be a triangle inscribed in a circle, where the tangents to the circle at points $B$ and $C$ intersect at the point $P$. Let $M$ be a point on the arc $AC$ (not containing $B$) such that $M \neq A$ and $M \neq C$. Let the lines $BC$ and $AM$ intersect at point $K$. Let $P'$ be the reflection of $P$ with respect to the line $AM$. The lines $AP'$ and $PM$ intersect at point $Q$, and $PM$ intersects the circumcircle of $\triangle ABC$ again at point $N$.

Prove that the point $Q$ lies on the circumcircle of $\triangle ANK$.
3 replies
youochange
Today at 11:27 AM
Double07
21 minutes ago
mdk2013
Mar 30, 2025
mdk2013
an hour ago
SuMAC Email
miguel00   6
N an hour ago by NoSignOfTheta
Did anyone get an email from SuMAC checking availability for summer camp you applied for (residential/online)? I don't know whether it is a good sign or just something that everyone got.
6 replies
+1 w
miguel00
Apr 2, 2025
NoSignOfTheta
an hour ago
Sums of pairs in a sequence
tenniskidperson3   56
N 2 hours ago by Marcus_Zhang
Source: USAJMO 2010, Problem 2
Let $n > 1$ be an integer. Find, with proof, all sequences $x_1 , x_2 , \ldots , x_{n-1}$ of positive integers with the following three properties:
(a). $x_1 < x_2 < \cdots < x_{n-1}$ ;
(b). $x_i + x_{n-i} = 2n$ for all $i = 1, 2, \ldots , n - 1$;
(c). given any two indices $i$ and $j$ (not necessarily distinct) for which $x_i + x_j < 2n$, there is an index $k$ such that $x_i + x_j = x_k$.
56 replies
tenniskidperson3
Apr 29, 2010
Marcus_Zhang
2 hours ago
USA(J)MO qualification
mathkidAP   23
N 4 hours ago by AbhayAttarde01
Hello. I am an 8th grade student who wants to make jmo or usamo. How much practice do i need for this? i have a 63 on amc 10b and i mock roughly 90-100s on most amc 10s.
23 replies
mathkidAP
Apr 4, 2025
AbhayAttarde01
4 hours ago
k Help me find this person..
Pomansq   6
N 5 hours ago by DottedCaculator
I know a certain person with the following achievements, and I think his name is Jonathan...

USAMO Silver, MOP Attendee
PRIMES attendee
USAPHO Qualifier
USABO Awardee
HMMT Awardee...
etc...

I believe I may have met him on a website with anonymous users, and he helped me with math questions when he was working under a certain pseudonym. Please let me know if anyone has his contact...
6 replies
Pomansq
Today at 3:51 AM
DottedCaculator
5 hours ago
Predicted AMC 8 Scores
megahertz13   141
N Today at 8:05 AM by fake123
$\begin{tabular}{c|c|c|c}Username & Grade & AMC8 Score \\ \hline
megahertz13 & 5 & 23 \\
\end{tabular}$
141 replies
megahertz13
Jan 25, 2024
fake123
Today at 8:05 AM
1989 AMC 12 #30 - Boys and Girls in a Line
dft   7
N Today at 6:56 AM by NicoN9
Suppose that $7$ boys and $13$ girls line up in a row. Let $S$ be the number of places in the row where a boy and a girl are standing next to each other. For example, for the row $GBBGGGBGBGGGBGBGGBGG$ we have $S=12$. The average value of $S$ (if all possible orders of the 20 people are considered) is closest to

$ \textbf{(A)}\ 9 \qquad\textbf{(B)}\ 10 \qquad\textbf{(C)}\ 11 \qquad\textbf{(D)}\ 12 \qquad\textbf{(E)}\ 13 $
7 replies
dft
Dec 31, 2011
NicoN9
Today at 6:56 AM
Catch those negatives
cappucher   43
N Today at 2:08 AM by sadas123
Source: 2024 AMC 10A P11
How many ordered pairs of integers $(m, n)$ satisfy $\sqrt{n^2 - 49} = m$?

$
\textbf{(A) }1 \qquad
\textbf{(B) }2 \qquad
\textbf{(C) }3 \qquad
\textbf{(D) }4 \qquad
\textbf{(E) } \text{Infinitely many} \qquad
$
43 replies
cappucher
Nov 7, 2024
sadas123
Today at 2:08 AM
2n equations
P_Groudon   80
N Yesterday at 7:27 PM by vincentwant
Let $n \geq 4$ be an integer. Find all positive real solutions to the following system of $2n$ equations:

\begin{align*}
a_{1} &=\frac{1}{a_{2 n}}+\frac{1}{a_{2}}, & a_{2}&=a_{1}+a_{3}, \\
a_{3}&=\frac{1}{a_{2}}+\frac{1}{a_{4}}, & a_{4}&=a_{3}+a_{5}, \\
a_{5}&=\frac{1}{a_{4}}+\frac{1}{a_{6}}, & a_{6}&=a_{5}+a_{7} \\
&\vdots & &\vdots \\
a_{2 n-1}&=\frac{1}{a_{2 n-2}}+\frac{1}{a_{2 n}}, & a_{2 n}&=a_{2 n-1}+a_{1}
\end{align*}
80 replies
P_Groudon
Apr 15, 2021
vincentwant
Yesterday at 7:27 PM
basic nt
zhoujef000   38
N Yesterday at 6:08 PM by Apple_maths60
Source: 2025 AIME I #1
Find the sum of all integer bases $b>9$ for which $17_b$ is a divisor of $97_b.$
38 replies
zhoujef000
Feb 7, 2025
Apple_maths60
Yesterday at 6:08 PM
2024 BxMO P3
beansenthusiast505   4
N Mar 30, 2025 by GeorgeMetrical123
Source: 2024 BxMO P3
Let $ABC$ be a triangle with incentre $I$ and circumcircle $\Omega$ such that $\left|AC\right|\neq\left|BC\right|$. The internal angle bisector of $\angle CAB$ intersects side $BC$ at $D$ and the external angle bisectors of $\angle ABC$ and $\angle BCA$ intersect $\Omega$ at $E$ and $F$ respectively. Let $G$ be the intersection of lines $AE$ and $FI$ and let $\Gamma$ be the circumcircle of triangle $BDI$. Show that $E$ lies on $\Gamma$ if and only if $G$ lies on $\Gamma$.
4 replies
beansenthusiast505
Apr 28, 2024
GeorgeMetrical123
Mar 30, 2025
2024 BxMO P3
G H J
G H BBookmark kLocked kLocked NReply
Source: 2024 BxMO P3
The post below has been deleted. Click to close.
This post has been deleted. Click here to see post.
beansenthusiast505
26 posts
#1 • 1 Y
Y by Rounak_iitr
Let $ABC$ be a triangle with incentre $I$ and circumcircle $\Omega$ such that $\left|AC\right|\neq\left|BC\right|$. The internal angle bisector of $\angle CAB$ intersects side $BC$ at $D$ and the external angle bisectors of $\angle ABC$ and $\angle BCA$ intersect $\Omega$ at $E$ and $F$ respectively. Let $G$ be the intersection of lines $AE$ and $FI$ and let $\Gamma$ be the circumcircle of triangle $BDI$. Show that $E$ lies on $\Gamma$ if and only if $G$ lies on $\Gamma$.
Z K Y
The post below has been deleted. Click to close.
This post has been deleted. Click here to see post.
sami1618
883 posts
#2
Y by
Whenever I draw it neither point lies on the circle
Z K Y
The post below has been deleted. Click to close.
This post has been deleted. Click here to see post.
YaoAOPS
1501 posts
#3
Y by
This took longer than I'd like. Nice-ish problem.


First suppose that $BEDI$ is cyclic. Then by radical axis on $(BEDI)$, $\Omega$ it follows that $IDFC$ is cyclic. Let $I_A$ be the $A$-excenter, note that $(BII_AC)$ is cyclic. Then note that $E - D - F$ are collinear since
\[ \measuredangle EDI = \measuredangle EBI = \measuredangle I_ABI = \measuredangle I_ACI = \measuredangle FCI = FDI. \]As such, $\measuredangle GED = \measuredangle AEF = \measuredangle ACF = \measuredangle I_ACB = \measuredangle FCD = \measuredangle FID$ which finishes.

Now consider the case where $BEGI$ is cyclic. Define $D' = (BGEI) \cap (ICF)$. Then by radaxis on $(BGEI)$, $\Omega$, and $(ICF)$, it follows that $D'$ is on $AI$. Then we can angle chase $E-D'-F$ again. Then by a similar angle chase we get $\measuredangle GID = \measuredangle FID = \measuredangle GED = \measuredangle FCB$.
Z K Y
The post below has been deleted. Click to close.
This post has been deleted. Click here to see post.
Euler365
142 posts
#4
Y by
Just note that both conditions are equivalent to $I$ being orthocentre of $\triangle AEF$ thru simple angle chase.
Z K Y
The post below has been deleted. Click to close.
This post has been deleted. Click here to see post.
GeorgeMetrical123
7 posts
#5
Y by
Suppose $(BEDI)$ is a cyclic quad. Then, by radical axis on $(BEDI)$ and $\Gamma$ we get that $I_A$ lies on the radical axis.From there, it follows that $ (IDCF) $ is a cyclic quad. From there, we angle chase that $\angle DEF = 0$:
\begin{align*}
\angle DEI_A &= 180^\circ - \angle BED \\
& = \angle BID \\
&= 180^\circ - \angle AIB \\ 
&= 180^\circ - (90^\circ + \gamma) \\
& = 90^\circ - \gamma \\
& = \angle BAF \\
& = 180^\circ - \angle BEF \\
&= \angle I_AEF
\end{align*}Where $\gamma = \frac{1}{2}\angle ACB $.
From there, we angle chase again:
\begin{align*}
\angle GEB  &= \angle AEB \\
&=  \angle ACB \\
&= \angle BCF - \angle BCT \\
&= \angle DCF - \angle BIT \\
&= 180^\circ - \angle DIF - \angle BID \\
&= 180^\circ - \angle BIF \\
&= \angle GIB
\end{align*}voila!

For the other part, we angle chase again. We will first prove that $ (AGDF)$ is a cyclic quad.
\begin{align*}
\angle FGD &= \angle IGD \\
&= \angle IBD \\
&= \beta \\
&= 90^\circ - \alpha - \gamma \\
&= \angle BAF - \angle BAD \\
&= \angle DAF 
\end{align*}Now, we prove that $(IDCF)$ is a cyclic quad and from there we finish like in the last proof.
\begin{align*}
\angle FID &= \angle GFD \\
&= \angle GAD \\
&= \angle EAD \\
&= \angle EAC - \angle DAC \\
&= 90^\circ - \beta - \alpha \\
&= \gamma \\
&= \angle ICB \\
&= \angle ICD
\end{align*}Which proves that $(IDCF)$ is a cyclic quad and by using the same method as above, we are finished! Q.E.D.
Z K Y
N Quick Reply
G
H
=
a